2016-2017/3e/DM/DM_17_04_05/pgm_calc.tex

77 lines
2.3 KiB
TeX

% theme: Programme de calculs
%- set a = randint(5, 15)
%- set b = randint(2, 15)
%- set c = b - 3
%- set d = a * b
Voici un programme de calcul.
\begin{center}
\fbox{\colorbox{base2}{
\begin{minipage}[h]{0.5\textwidth}
Prendre un nombre\\
Lui ajouter \Var{a} \\
Multiplier le résultat par \Var{b}\\
Enlever \Var{c} fois le nombre de départ\\
Enlever \Var{d}
\end{minipage}
}}
\end{center}
\begin{enumerate}
%- set choix1 = randint(5, 20)
%- set ans1 = choix1 * 3
\item Vérifier quand quand on choisit \Var{choix1} on obtient \Var{ans1} à la fin.
\begin{solution}
\begin{eqnarray*}
\Var{choix1} \rightarrow
%- set e1 = choix1 + a
\Var{e1} \rightarrow
%- set e2 = e1 * a
\Var{e2} \rightarrow
%- set e3 = e2 - choix1 * c
\Var{e3} \rightarrow
%- set e4 = e2 - d
\Var{e4}
\end{eqnarray*}
\end{solution}
%- set choix2 = randint(5, 20)
%- set ans2 = choix2 * 3
%- if choix1 == choix2
%- set choix2 = -choix1
%- set ans2 = choix2 * 3
%- endif
\item Quel nombre obtient-on quand on choisit \Var{choix2}?
\begin{solution}
\begin{eqnarray*}
\Var{choix2} \rightarrow
%- set e1 = choix1 + a
\Var{e1} \rightarrow
%- set e2 = e1 * b
\Var{e2} \rightarrow
%- set e3 = e2 - choix1 * c
\Var{e3} \rightarrow
%- set e4 = e2 - d
\Var{e4}
\end{eqnarray*}
\end{solution}
\item Démontrer que quand on applique le programme à $x$, on obtient
\begin{eqnarray*}
\Var{b}(x + \Var{a}) - \Var{c}x - \Var{d}
\end{eqnarray*}
%- set end3 = randint(20, 40)
%- if not (end3 % 3)
%- set end3 = end3 + 1
%- endif
\item Quel nombre doit-on choisir pour obtenir \Var{end3} à la fin?
\begin{solution}
On doit choisir $\dfrac{\Var{end3}}{3}$
\end{solution}
\item Démontrer que pour n'importe quel nombre choisi, on obtient toujours le triple de ce nombre.
\end{enumerate}